Monotone matrix norms











up vote
12
down vote

favorite
3












[Ciarlet 2.2-10]




  1. Let $mathscr{S}_n$ be the set of symmetric matrices and $mathscr{S}_n^+$ the subset of non-negative definite symmetric matrices. A matrix norm $|cdot|$ to be monotone if
    $$Ainmathscr{S}_n^+; wedge; B-Ainmathscr{S}_n^+ Rightarrow |A| leq |B|.$$
    Show that the norms $|cdot|_2$ and $|cdot|_F$ (Frobenus norm) are monotone.

  2. More generally, show that if a matrix norm $|cdot|$ is invariant under unitary transformations, that is, if $|A| = |AU| = |UA|$ for every unitary matrix $U$, then it is monotone.

  3. Let $|cdot|$ be a monotone norm and $mbox{cond}(cdot)$ the condition number function associated with it. Prove that
    $$A,Binmathscr{S}_n^* Rightarrow mbox{cond}(A+B) leq maxleft{mbox{cond}(A),; mbox{cond}(B)right}$$
    where $mathscr{S}_n^*$ denotes the subset of positive definite symmetric matrices.




I already have proved (1), and I proved that $lambda_i(A) leq lambda_i(B)$, $forall i=1,2,ldots,n$ and $forall A,B-Ainmathscr{S}_n^+$. But I have had problems in order to prove (2) and (3). For (2), i proved that
begin{eqnarray*}
|A| & = & |U^*AU| = |mbox{diag}(lambda_i(A))|,\[0.3cm]
|B| & = & |V^*BV| = |mbox{diag}(lambda_i(B))|.
end{eqnarray*}
but I don't know what I should do next. Please help me and thanks in advance.










share|cite|improve this question




























    up vote
    12
    down vote

    favorite
    3












    [Ciarlet 2.2-10]




    1. Let $mathscr{S}_n$ be the set of symmetric matrices and $mathscr{S}_n^+$ the subset of non-negative definite symmetric matrices. A matrix norm $|cdot|$ to be monotone if
      $$Ainmathscr{S}_n^+; wedge; B-Ainmathscr{S}_n^+ Rightarrow |A| leq |B|.$$
      Show that the norms $|cdot|_2$ and $|cdot|_F$ (Frobenus norm) are monotone.

    2. More generally, show that if a matrix norm $|cdot|$ is invariant under unitary transformations, that is, if $|A| = |AU| = |UA|$ for every unitary matrix $U$, then it is monotone.

    3. Let $|cdot|$ be a monotone norm and $mbox{cond}(cdot)$ the condition number function associated with it. Prove that
      $$A,Binmathscr{S}_n^* Rightarrow mbox{cond}(A+B) leq maxleft{mbox{cond}(A),; mbox{cond}(B)right}$$
      where $mathscr{S}_n^*$ denotes the subset of positive definite symmetric matrices.




    I already have proved (1), and I proved that $lambda_i(A) leq lambda_i(B)$, $forall i=1,2,ldots,n$ and $forall A,B-Ainmathscr{S}_n^+$. But I have had problems in order to prove (2) and (3). For (2), i proved that
    begin{eqnarray*}
    |A| & = & |U^*AU| = |mbox{diag}(lambda_i(A))|,\[0.3cm]
    |B| & = & |V^*BV| = |mbox{diag}(lambda_i(B))|.
    end{eqnarray*}
    but I don't know what I should do next. Please help me and thanks in advance.










    share|cite|improve this question


























      up vote
      12
      down vote

      favorite
      3









      up vote
      12
      down vote

      favorite
      3






      3





      [Ciarlet 2.2-10]




      1. Let $mathscr{S}_n$ be the set of symmetric matrices and $mathscr{S}_n^+$ the subset of non-negative definite symmetric matrices. A matrix norm $|cdot|$ to be monotone if
        $$Ainmathscr{S}_n^+; wedge; B-Ainmathscr{S}_n^+ Rightarrow |A| leq |B|.$$
        Show that the norms $|cdot|_2$ and $|cdot|_F$ (Frobenus norm) are monotone.

      2. More generally, show that if a matrix norm $|cdot|$ is invariant under unitary transformations, that is, if $|A| = |AU| = |UA|$ for every unitary matrix $U$, then it is monotone.

      3. Let $|cdot|$ be a monotone norm and $mbox{cond}(cdot)$ the condition number function associated with it. Prove that
        $$A,Binmathscr{S}_n^* Rightarrow mbox{cond}(A+B) leq maxleft{mbox{cond}(A),; mbox{cond}(B)right}$$
        where $mathscr{S}_n^*$ denotes the subset of positive definite symmetric matrices.




      I already have proved (1), and I proved that $lambda_i(A) leq lambda_i(B)$, $forall i=1,2,ldots,n$ and $forall A,B-Ainmathscr{S}_n^+$. But I have had problems in order to prove (2) and (3). For (2), i proved that
      begin{eqnarray*}
      |A| & = & |U^*AU| = |mbox{diag}(lambda_i(A))|,\[0.3cm]
      |B| & = & |V^*BV| = |mbox{diag}(lambda_i(B))|.
      end{eqnarray*}
      but I don't know what I should do next. Please help me and thanks in advance.










      share|cite|improve this question















      [Ciarlet 2.2-10]




      1. Let $mathscr{S}_n$ be the set of symmetric matrices and $mathscr{S}_n^+$ the subset of non-negative definite symmetric matrices. A matrix norm $|cdot|$ to be monotone if
        $$Ainmathscr{S}_n^+; wedge; B-Ainmathscr{S}_n^+ Rightarrow |A| leq |B|.$$
        Show that the norms $|cdot|_2$ and $|cdot|_F$ (Frobenus norm) are monotone.

      2. More generally, show that if a matrix norm $|cdot|$ is invariant under unitary transformations, that is, if $|A| = |AU| = |UA|$ for every unitary matrix $U$, then it is monotone.

      3. Let $|cdot|$ be a monotone norm and $mbox{cond}(cdot)$ the condition number function associated with it. Prove that
        $$A,Binmathscr{S}_n^* Rightarrow mbox{cond}(A+B) leq maxleft{mbox{cond}(A),; mbox{cond}(B)right}$$
        where $mathscr{S}_n^*$ denotes the subset of positive definite symmetric matrices.




      I already have proved (1), and I proved that $lambda_i(A) leq lambda_i(B)$, $forall i=1,2,ldots,n$ and $forall A,B-Ainmathscr{S}_n^+$. But I have had problems in order to prove (2) and (3). For (2), i proved that
      begin{eqnarray*}
      |A| & = & |U^*AU| = |mbox{diag}(lambda_i(A))|,\[0.3cm]
      |B| & = & |V^*BV| = |mbox{diag}(lambda_i(B))|.
      end{eqnarray*}
      but I don't know what I should do next. Please help me and thanks in advance.







      linear-algebra norm






      share|cite|improve this question















      share|cite|improve this question













      share|cite|improve this question




      share|cite|improve this question








      edited Feb 26 '17 at 3:21









      Martin Argerami

      121k1073172




      121k1073172










      asked Apr 27 '13 at 17:56









      FASCH

      642926




      642926






















          1 Answer
          1






          active

          oldest

          votes

















          up vote
          0
          down vote













          For part 2:



          To simplify notation, define a function $g:mathbb R^ntomathbb R$ by $g(x_1,ldots,x_n)=|text{diag}(x_j)|$. Since $|cdot|$ is a unitarily invariant matrix norm, we have that




          • $g$ is a norm on $mathbb R^n$


          • $g(x_1,ldots,x_n)=g(|x_1|,ldots,|x_n|)$ (this comes from the unitary invariance)


          • $g(x_1,ldots,x_n)=g(x_{sigma(1)},ldots,x_{sigma(n)})$ for any permutation $sigma$.



          Such a $g$ is called a gauge function.



          Now, if $tin[0,1]$, then (writing $x=(x_1,ldots,x_n)$)
          begin{align}
          g(tx_1,x_2,ldots,x_n)&=gleft(frac{1+t}2,x+frac{1-t}2(-x_1,x_2,ldots,x_nright)\ \
          &leqfrac{1+t}2,g(x)+frac{1-t}2g(-x_1,x_2,ldots,x_n)\ \
          &=frac{1+t}2g(x)+frac{1-t}2g(x)=g(x).
          end{align}
          Applying the above inductively, we get
          $$
          g(t_1x_1,ldots,t_nx_n)leq g(x)
          $$
          whenever $t_1,ldots,t_nin [0,1]$.



          Since $0leqlambda_j(A)leqlambda_j(B)$ for all $j$, we have $lambda_j(A)=t_jlambda_j(B)$ for appropriate $t_1,ldots,t_nin[0,1]$.
          Then
          begin{align}
          |A|&=|text{diag}(lambda_j(A))|=|text{diag}(t_j,lambda_j(B))|\ \
          &leq |text{diag}(lambda_j(B))|=|B|
          end{align}



          For part 3, I know of the original proof by Marshall and Olkin (1973). Assume $text{cond}(A)leqtext{cond}(B)$. Let $A'=A/|A|$, $B'=B/|B|$, and $t=frac{|A|}{|A|+|B|}$.



          We have, in the new notation, that $$tag{0}|(A')^{-1}|leq|(B')^{-1}|.$$ And
          $$tag{1}
          |tA'+(1-t)B'|leq t|A'|+(1-t)|B'|=1.
          $$
          Also, as the inverse is convex on the set of positive-definite matrices,
          $$tag{2}
          (tA'+(1-t)B')^{-1}leq t(A')^{-1}+(1-t)(B')^{-1}.
          $$
          Thus, using $(2)$ and $(0)$,
          begin{align}tag{3}
          |(tA'+(1-t)B')^{-1}|&leq|t(A')^{-1}+(1-t)(B')^{-1}|\
          &leq t|(A')^{-1}|+(1-t)|(B')^{-1}|\ &leq|(B')^{-1}|
          end{align}
          Now, combining $(1)$, and $(3)$,
          begin{align}tag{4}
          |tA'+(1-t)B'|,|(tA'+(1-t)B')^{-1}|&leq |(tA'+(1-t)B')^{-1}|leq|(B')^{-1}|.
          end{align}
          If we now use the definitions of $A'$ and $B'$, we get
          $$
          tA'+(1-t)B'=frac1{|A|+|B|},left(A+Bright), (B')^{-1}=|B|,B^{-1}.
          $$
          We may thus rewrite $(4)$ as
          $$
          |A+B|,|(A+B)^{-1}|leq |B|,|B^{-1}|,
          $$
          as desired.






          share|cite|improve this answer























            Your Answer





            StackExchange.ifUsing("editor", function () {
            return StackExchange.using("mathjaxEditing", function () {
            StackExchange.MarkdownEditor.creationCallbacks.add(function (editor, postfix) {
            StackExchange.mathjaxEditing.prepareWmdForMathJax(editor, postfix, [["$", "$"], ["\\(","\\)"]]);
            });
            });
            }, "mathjax-editing");

            StackExchange.ready(function() {
            var channelOptions = {
            tags: "".split(" "),
            id: "69"
            };
            initTagRenderer("".split(" "), "".split(" "), channelOptions);

            StackExchange.using("externalEditor", function() {
            // Have to fire editor after snippets, if snippets enabled
            if (StackExchange.settings.snippets.snippetsEnabled) {
            StackExchange.using("snippets", function() {
            createEditor();
            });
            }
            else {
            createEditor();
            }
            });

            function createEditor() {
            StackExchange.prepareEditor({
            heartbeatType: 'answer',
            convertImagesToLinks: true,
            noModals: true,
            showLowRepImageUploadWarning: true,
            reputationToPostImages: 10,
            bindNavPrevention: true,
            postfix: "",
            imageUploader: {
            brandingHtml: "Powered by u003ca class="icon-imgur-white" href="https://imgur.com/"u003eu003c/au003e",
            contentPolicyHtml: "User contributions licensed under u003ca href="https://creativecommons.org/licenses/by-sa/3.0/"u003ecc by-sa 3.0 with attribution requiredu003c/au003e u003ca href="https://stackoverflow.com/legal/content-policy"u003e(content policy)u003c/au003e",
            allowUrls: true
            },
            noCode: true, onDemand: true,
            discardSelector: ".discard-answer"
            ,immediatelyShowMarkdownHelp:true
            });


            }
            });














             

            draft saved


            draft discarded


















            StackExchange.ready(
            function () {
            StackExchange.openid.initPostLogin('.new-post-login', 'https%3a%2f%2fmath.stackexchange.com%2fquestions%2f374500%2fmonotone-matrix-norms%23new-answer', 'question_page');
            }
            );

            Post as a guest















            Required, but never shown

























            1 Answer
            1






            active

            oldest

            votes








            1 Answer
            1






            active

            oldest

            votes









            active

            oldest

            votes






            active

            oldest

            votes








            up vote
            0
            down vote













            For part 2:



            To simplify notation, define a function $g:mathbb R^ntomathbb R$ by $g(x_1,ldots,x_n)=|text{diag}(x_j)|$. Since $|cdot|$ is a unitarily invariant matrix norm, we have that




            • $g$ is a norm on $mathbb R^n$


            • $g(x_1,ldots,x_n)=g(|x_1|,ldots,|x_n|)$ (this comes from the unitary invariance)


            • $g(x_1,ldots,x_n)=g(x_{sigma(1)},ldots,x_{sigma(n)})$ for any permutation $sigma$.



            Such a $g$ is called a gauge function.



            Now, if $tin[0,1]$, then (writing $x=(x_1,ldots,x_n)$)
            begin{align}
            g(tx_1,x_2,ldots,x_n)&=gleft(frac{1+t}2,x+frac{1-t}2(-x_1,x_2,ldots,x_nright)\ \
            &leqfrac{1+t}2,g(x)+frac{1-t}2g(-x_1,x_2,ldots,x_n)\ \
            &=frac{1+t}2g(x)+frac{1-t}2g(x)=g(x).
            end{align}
            Applying the above inductively, we get
            $$
            g(t_1x_1,ldots,t_nx_n)leq g(x)
            $$
            whenever $t_1,ldots,t_nin [0,1]$.



            Since $0leqlambda_j(A)leqlambda_j(B)$ for all $j$, we have $lambda_j(A)=t_jlambda_j(B)$ for appropriate $t_1,ldots,t_nin[0,1]$.
            Then
            begin{align}
            |A|&=|text{diag}(lambda_j(A))|=|text{diag}(t_j,lambda_j(B))|\ \
            &leq |text{diag}(lambda_j(B))|=|B|
            end{align}



            For part 3, I know of the original proof by Marshall and Olkin (1973). Assume $text{cond}(A)leqtext{cond}(B)$. Let $A'=A/|A|$, $B'=B/|B|$, and $t=frac{|A|}{|A|+|B|}$.



            We have, in the new notation, that $$tag{0}|(A')^{-1}|leq|(B')^{-1}|.$$ And
            $$tag{1}
            |tA'+(1-t)B'|leq t|A'|+(1-t)|B'|=1.
            $$
            Also, as the inverse is convex on the set of positive-definite matrices,
            $$tag{2}
            (tA'+(1-t)B')^{-1}leq t(A')^{-1}+(1-t)(B')^{-1}.
            $$
            Thus, using $(2)$ and $(0)$,
            begin{align}tag{3}
            |(tA'+(1-t)B')^{-1}|&leq|t(A')^{-1}+(1-t)(B')^{-1}|\
            &leq t|(A')^{-1}|+(1-t)|(B')^{-1}|\ &leq|(B')^{-1}|
            end{align}
            Now, combining $(1)$, and $(3)$,
            begin{align}tag{4}
            |tA'+(1-t)B'|,|(tA'+(1-t)B')^{-1}|&leq |(tA'+(1-t)B')^{-1}|leq|(B')^{-1}|.
            end{align}
            If we now use the definitions of $A'$ and $B'$, we get
            $$
            tA'+(1-t)B'=frac1{|A|+|B|},left(A+Bright), (B')^{-1}=|B|,B^{-1}.
            $$
            We may thus rewrite $(4)$ as
            $$
            |A+B|,|(A+B)^{-1}|leq |B|,|B^{-1}|,
            $$
            as desired.






            share|cite|improve this answer



























              up vote
              0
              down vote













              For part 2:



              To simplify notation, define a function $g:mathbb R^ntomathbb R$ by $g(x_1,ldots,x_n)=|text{diag}(x_j)|$. Since $|cdot|$ is a unitarily invariant matrix norm, we have that




              • $g$ is a norm on $mathbb R^n$


              • $g(x_1,ldots,x_n)=g(|x_1|,ldots,|x_n|)$ (this comes from the unitary invariance)


              • $g(x_1,ldots,x_n)=g(x_{sigma(1)},ldots,x_{sigma(n)})$ for any permutation $sigma$.



              Such a $g$ is called a gauge function.



              Now, if $tin[0,1]$, then (writing $x=(x_1,ldots,x_n)$)
              begin{align}
              g(tx_1,x_2,ldots,x_n)&=gleft(frac{1+t}2,x+frac{1-t}2(-x_1,x_2,ldots,x_nright)\ \
              &leqfrac{1+t}2,g(x)+frac{1-t}2g(-x_1,x_2,ldots,x_n)\ \
              &=frac{1+t}2g(x)+frac{1-t}2g(x)=g(x).
              end{align}
              Applying the above inductively, we get
              $$
              g(t_1x_1,ldots,t_nx_n)leq g(x)
              $$
              whenever $t_1,ldots,t_nin [0,1]$.



              Since $0leqlambda_j(A)leqlambda_j(B)$ for all $j$, we have $lambda_j(A)=t_jlambda_j(B)$ for appropriate $t_1,ldots,t_nin[0,1]$.
              Then
              begin{align}
              |A|&=|text{diag}(lambda_j(A))|=|text{diag}(t_j,lambda_j(B))|\ \
              &leq |text{diag}(lambda_j(B))|=|B|
              end{align}



              For part 3, I know of the original proof by Marshall and Olkin (1973). Assume $text{cond}(A)leqtext{cond}(B)$. Let $A'=A/|A|$, $B'=B/|B|$, and $t=frac{|A|}{|A|+|B|}$.



              We have, in the new notation, that $$tag{0}|(A')^{-1}|leq|(B')^{-1}|.$$ And
              $$tag{1}
              |tA'+(1-t)B'|leq t|A'|+(1-t)|B'|=1.
              $$
              Also, as the inverse is convex on the set of positive-definite matrices,
              $$tag{2}
              (tA'+(1-t)B')^{-1}leq t(A')^{-1}+(1-t)(B')^{-1}.
              $$
              Thus, using $(2)$ and $(0)$,
              begin{align}tag{3}
              |(tA'+(1-t)B')^{-1}|&leq|t(A')^{-1}+(1-t)(B')^{-1}|\
              &leq t|(A')^{-1}|+(1-t)|(B')^{-1}|\ &leq|(B')^{-1}|
              end{align}
              Now, combining $(1)$, and $(3)$,
              begin{align}tag{4}
              |tA'+(1-t)B'|,|(tA'+(1-t)B')^{-1}|&leq |(tA'+(1-t)B')^{-1}|leq|(B')^{-1}|.
              end{align}
              If we now use the definitions of $A'$ and $B'$, we get
              $$
              tA'+(1-t)B'=frac1{|A|+|B|},left(A+Bright), (B')^{-1}=|B|,B^{-1}.
              $$
              We may thus rewrite $(4)$ as
              $$
              |A+B|,|(A+B)^{-1}|leq |B|,|B^{-1}|,
              $$
              as desired.






              share|cite|improve this answer

























                up vote
                0
                down vote










                up vote
                0
                down vote









                For part 2:



                To simplify notation, define a function $g:mathbb R^ntomathbb R$ by $g(x_1,ldots,x_n)=|text{diag}(x_j)|$. Since $|cdot|$ is a unitarily invariant matrix norm, we have that




                • $g$ is a norm on $mathbb R^n$


                • $g(x_1,ldots,x_n)=g(|x_1|,ldots,|x_n|)$ (this comes from the unitary invariance)


                • $g(x_1,ldots,x_n)=g(x_{sigma(1)},ldots,x_{sigma(n)})$ for any permutation $sigma$.



                Such a $g$ is called a gauge function.



                Now, if $tin[0,1]$, then (writing $x=(x_1,ldots,x_n)$)
                begin{align}
                g(tx_1,x_2,ldots,x_n)&=gleft(frac{1+t}2,x+frac{1-t}2(-x_1,x_2,ldots,x_nright)\ \
                &leqfrac{1+t}2,g(x)+frac{1-t}2g(-x_1,x_2,ldots,x_n)\ \
                &=frac{1+t}2g(x)+frac{1-t}2g(x)=g(x).
                end{align}
                Applying the above inductively, we get
                $$
                g(t_1x_1,ldots,t_nx_n)leq g(x)
                $$
                whenever $t_1,ldots,t_nin [0,1]$.



                Since $0leqlambda_j(A)leqlambda_j(B)$ for all $j$, we have $lambda_j(A)=t_jlambda_j(B)$ for appropriate $t_1,ldots,t_nin[0,1]$.
                Then
                begin{align}
                |A|&=|text{diag}(lambda_j(A))|=|text{diag}(t_j,lambda_j(B))|\ \
                &leq |text{diag}(lambda_j(B))|=|B|
                end{align}



                For part 3, I know of the original proof by Marshall and Olkin (1973). Assume $text{cond}(A)leqtext{cond}(B)$. Let $A'=A/|A|$, $B'=B/|B|$, and $t=frac{|A|}{|A|+|B|}$.



                We have, in the new notation, that $$tag{0}|(A')^{-1}|leq|(B')^{-1}|.$$ And
                $$tag{1}
                |tA'+(1-t)B'|leq t|A'|+(1-t)|B'|=1.
                $$
                Also, as the inverse is convex on the set of positive-definite matrices,
                $$tag{2}
                (tA'+(1-t)B')^{-1}leq t(A')^{-1}+(1-t)(B')^{-1}.
                $$
                Thus, using $(2)$ and $(0)$,
                begin{align}tag{3}
                |(tA'+(1-t)B')^{-1}|&leq|t(A')^{-1}+(1-t)(B')^{-1}|\
                &leq t|(A')^{-1}|+(1-t)|(B')^{-1}|\ &leq|(B')^{-1}|
                end{align}
                Now, combining $(1)$, and $(3)$,
                begin{align}tag{4}
                |tA'+(1-t)B'|,|(tA'+(1-t)B')^{-1}|&leq |(tA'+(1-t)B')^{-1}|leq|(B')^{-1}|.
                end{align}
                If we now use the definitions of $A'$ and $B'$, we get
                $$
                tA'+(1-t)B'=frac1{|A|+|B|},left(A+Bright), (B')^{-1}=|B|,B^{-1}.
                $$
                We may thus rewrite $(4)$ as
                $$
                |A+B|,|(A+B)^{-1}|leq |B|,|B^{-1}|,
                $$
                as desired.






                share|cite|improve this answer














                For part 2:



                To simplify notation, define a function $g:mathbb R^ntomathbb R$ by $g(x_1,ldots,x_n)=|text{diag}(x_j)|$. Since $|cdot|$ is a unitarily invariant matrix norm, we have that




                • $g$ is a norm on $mathbb R^n$


                • $g(x_1,ldots,x_n)=g(|x_1|,ldots,|x_n|)$ (this comes from the unitary invariance)


                • $g(x_1,ldots,x_n)=g(x_{sigma(1)},ldots,x_{sigma(n)})$ for any permutation $sigma$.



                Such a $g$ is called a gauge function.



                Now, if $tin[0,1]$, then (writing $x=(x_1,ldots,x_n)$)
                begin{align}
                g(tx_1,x_2,ldots,x_n)&=gleft(frac{1+t}2,x+frac{1-t}2(-x_1,x_2,ldots,x_nright)\ \
                &leqfrac{1+t}2,g(x)+frac{1-t}2g(-x_1,x_2,ldots,x_n)\ \
                &=frac{1+t}2g(x)+frac{1-t}2g(x)=g(x).
                end{align}
                Applying the above inductively, we get
                $$
                g(t_1x_1,ldots,t_nx_n)leq g(x)
                $$
                whenever $t_1,ldots,t_nin [0,1]$.



                Since $0leqlambda_j(A)leqlambda_j(B)$ for all $j$, we have $lambda_j(A)=t_jlambda_j(B)$ for appropriate $t_1,ldots,t_nin[0,1]$.
                Then
                begin{align}
                |A|&=|text{diag}(lambda_j(A))|=|text{diag}(t_j,lambda_j(B))|\ \
                &leq |text{diag}(lambda_j(B))|=|B|
                end{align}



                For part 3, I know of the original proof by Marshall and Olkin (1973). Assume $text{cond}(A)leqtext{cond}(B)$. Let $A'=A/|A|$, $B'=B/|B|$, and $t=frac{|A|}{|A|+|B|}$.



                We have, in the new notation, that $$tag{0}|(A')^{-1}|leq|(B')^{-1}|.$$ And
                $$tag{1}
                |tA'+(1-t)B'|leq t|A'|+(1-t)|B'|=1.
                $$
                Also, as the inverse is convex on the set of positive-definite matrices,
                $$tag{2}
                (tA'+(1-t)B')^{-1}leq t(A')^{-1}+(1-t)(B')^{-1}.
                $$
                Thus, using $(2)$ and $(0)$,
                begin{align}tag{3}
                |(tA'+(1-t)B')^{-1}|&leq|t(A')^{-1}+(1-t)(B')^{-1}|\
                &leq t|(A')^{-1}|+(1-t)|(B')^{-1}|\ &leq|(B')^{-1}|
                end{align}
                Now, combining $(1)$, and $(3)$,
                begin{align}tag{4}
                |tA'+(1-t)B'|,|(tA'+(1-t)B')^{-1}|&leq |(tA'+(1-t)B')^{-1}|leq|(B')^{-1}|.
                end{align}
                If we now use the definitions of $A'$ and $B'$, we get
                $$
                tA'+(1-t)B'=frac1{|A|+|B|},left(A+Bright), (B')^{-1}=|B|,B^{-1}.
                $$
                We may thus rewrite $(4)$ as
                $$
                |A+B|,|(A+B)^{-1}|leq |B|,|B^{-1}|,
                $$
                as desired.







                share|cite|improve this answer














                share|cite|improve this answer



                share|cite|improve this answer








                edited Apr 13 '17 at 12:21









                Community

                1




                1










                answered Feb 26 '17 at 3:21









                Martin Argerami

                121k1073172




                121k1073172






























                     

                    draft saved


                    draft discarded



















































                     


                    draft saved


                    draft discarded














                    StackExchange.ready(
                    function () {
                    StackExchange.openid.initPostLogin('.new-post-login', 'https%3a%2f%2fmath.stackexchange.com%2fquestions%2f374500%2fmonotone-matrix-norms%23new-answer', 'question_page');
                    }
                    );

                    Post as a guest















                    Required, but never shown





















































                    Required, but never shown














                    Required, but never shown












                    Required, but never shown







                    Required, but never shown

































                    Required, but never shown














                    Required, but never shown












                    Required, but never shown







                    Required, but never shown







                    Popular posts from this blog

                    To store a contact into the json file from server.js file using a class in NodeJS

                    Redirect URL with Chrome Remote Debugging Android Devices

                    Dieringhausen